4
$\begingroup$

Consider the following cooperative game played on the circle $S^1$, which we identify with $[0, 1]$ with its endpoints identified.

Let $0 < \alpha < 1$ be an irrational number. A total of $N \geq 2$ players each start at $0$. At every turn, the players each simultaneously bet heads or tails on a coin flip. If their choice matches, they move a distance of $\alpha$ clockwise, otherwise they move $\alpha$ counterclockwise.

A strategy is a sequence of heads/tails choices by the players that can depend on previous choices and coin flips, but not on current or future coin flips.

Question: Does there exist a strategy such that, almost surely the sequence of players’ positions is dense in $(S^1)^N$?

$\endgroup$
5
  • $\begingroup$ I'm not quite seeing how this is cooperative. $\endgroup$ Commented Mar 15 at 10:02
  • $\begingroup$ Some ideas, assuming all of them bet on a single coin flip (I did not check them/can try to write them later if noone answers): I believe a set of measure $1$ of the set of all strategies $(\{0,1\}^N)^{\mathbb{N}}$ which do not depend on previous coin flips should work. One may also be able to take any finite sequence of vectors $v_1,\dots,v_k$ in $\{-\alpha,\alpha\}^N$ that generate $\mathbb{R}^N$ as an $\mathbb{R}^N$-vector space (so $k\geq N$), and choose them as bets in repeat $\endgroup$ Commented Mar 15 at 13:05
  • $\begingroup$ @user479223 It is cooperative in the sense that the players are trying to cooperate to make the sequence dense (and they can coordinate guesses). $\endgroup$ Commented Mar 16 at 2:33
  • $\begingroup$ I just mean I don't see how cooperation helps - how does one player's choices affect the other players? I am just dumb, please forgive me. $\endgroup$ Commented Mar 16 at 10:44
  • $\begingroup$ @user479223 no you aren’t :P. Maybe this is the part you were missing (and i should’ve made more clear) - they all bet on the same coin. $\endgroup$ Commented Mar 16 at 10:46

2 Answers 2

5
$\begingroup$

Sure, let us take a sequence $v_{n, 1}, v_{n, 2},\ldots, v_{n, M(n)}$ of vectors in $\mathbb{R}^N$ such that each vector has coordinates $\pm \alpha$ and the fractional parts of partial sums of these vectors form $\frac{1}{n}$-net on $(S^1)^N$. Such a sequence exists for the same reason there is a robot which can solve every maze of size $n\times n$ -- just subdivide $(S^1)^N$ into finitely many parts, each of which has diameter smaller than $\frac{1}{2n}$, and then aim to visit each of them one by one (we can in two vectors shift any coordinate by $2\alpha$ while leaving each other fixed, and then each irrational rotation is dense. I think with this idea we can make what we want more explicit, by it is not the intuition I aim for).

Now just always bet this sequence of coinflips on repeat ($k$'th person bets tails if the corresponding coordinate is $+\alpha$ and heads if it is $-\alpha$). Sooner or later we will get all our guesses as we want, and regardless of the point where we start our points will form a $\frac{1}{n}$-net of the torus. Once this is achieved, go to the next $n$.

With a bit more suffering I think it is not hard to make the strategy independent of the previous coin flips (i.e. repeat $n$'th part of the strategy $K(n)$ many times where $K(n)$ is big enough, then we will get $\frac{1}{n}$-net for infinitely many $n$, hence we will have denseness).

$\endgroup$
6
  • 1
    $\begingroup$ Isn't what you mention in the first two lines impossible for big $n$, due to the set $\{-\alpha,\alpha\}^N$ being finite? $\endgroup$ Commented Mar 15 at 13:06
  • $\begingroup$ Doesn't sound like much suffering, the expected time to find a net is $2^{M(n)}$ repeats, it's just a Bernoulli process. $\endgroup$ Commented Mar 15 at 14:19
  • $\begingroup$ @VilleSalo it is indeed not much, but using even a single brain cell more than is required by the question is pain:) $\endgroup$ Commented Mar 15 at 14:29
  • $\begingroup$ @SaúlRM I have no idea what you mean, sure, $\{-\alpha, \alpha\}^N$ is finite, but we have here $\{-\alpha, \alpha\}^{NM(n)}$, which can be arbitrarily large. The only way I see is if for some reason you interpreted partial sums as partial sums of entries of each vector, and not of the vectors themselves ($v_{n, 1}$, $v_{n, 1}+v_{n, 2}$, $v_{n, 1}+v_{n,2}+v_{n,3},\ldots$) but then you would have had a compilation error couple of words later, as the partial sums of entries of a vector are elements of $S^1$ and not of $(S^1)^N$. $\endgroup$ Commented Mar 15 at 14:31
  • $\begingroup$ @AlekseiKulikov thanks. I somehow missed the "partial sums" part. Then my answer is similar to this, although the implementation is a bit different (which also makes it periodic and independent of previous flips), so I will leave it uploaded $\endgroup$ Commented Mar 15 at 14:40
3
$\begingroup$

Here is a strategy which is periodic and independent of previous flips.

As in Aleksei Kulikov's answer, we identify heads with $\alpha$ and tails with $-\alpha$, so that a bet is an element of $\{-\alpha,\alpha\}^N\subseteq\mathbb{R}^N$. Let $(e_i)_{i=1}^N$ be the usual basis of $\mathbb{R}^N$.

Consider the choices $v_1,\dots,v_n$ where $v_i=\alpha\sum_{j=1}^Ne_j-2\alpha e_i$. We will repeat the bets $v_1,v_2,\dots,v_n$ periodically, with period $n$. As $(\mathbb{S}^1)^N$ is second countable, we just need to check that, given some nonempty open set $O\subseteq(\mathbb{S}^1)^N$, $O$ is visited by the players with probability $1$.

To see why, note that the position of our players in $(\mathbb{S}^1)^N$ at time $n$ is given by $\pi\left(\sum_{i=1}^Na_i(n)v_i\right)$, where $\pi:\mathbb{R}^N\to(\mathbb{S}^1)^N$ is taking modulo $1$ in all coordinates, and $a_i(n)$ are integer values such that, from step $n-1$ to step $n$, we add $\pm 1$ to $a_{n\text{ mod }N}$ and keep the other coefficients the same.

So we can see the problem as a sequence of vectors (a random walk) $u(n)=(a_1(n),\dots,a_N(n))_{n\in\mathbb{N}}$ in $\mathbb{Z}^N$, where at each step we add $\pm e_{n\text{ mod }N}$. Let $$P:\mathbb{Z}^N\to(\mathbb{S}^1)^N;(a_1,\dots,a_N)\mapsto\pi\left(\sum_{i=1}^Na_iv_i\right)$$

Claim: Let $A=\{u\in\mathbb{Z}^N;P(u)\in O\}\subseteq\mathbb{Z}^N$. Then there is a finite set $F$ such that $A$ intersects $u+F$ for all $u\in\mathbb{Z}^N$.

Proof: Let $\varepsilon>0$ be such that $O$ contains an $\varepsilon$-ball, and choose the set $F$ such that $P(F)$ is $\varepsilon$-dense in $(\mathbb{S}^1)^N$. We are using here that $P(\mathbb{Z}^N)$ is dense in $(\mathbb{S}^1)^N$. $\square$

The claim implies that there is $\varepsilon>0$ and $L\in\mathbb{N}$ such that, for any $n$ and any initial position $u(n)$, the probability that at least one of the vectors $u(n+1),\dots,u(n+L)$ is inside $A$ is $>\varepsilon$ (e.g. let $L=n\cdot M$, where $M$ is the maximum distance between $0$ and any point of $F$).

Thus, for each $k$, the probability that no vector $u(n)$, for $n=1,\dots,kL$, is in $A$, is at most $(1-\varepsilon)^k\to0$. So almost surely $u(n)$ is in $A$ for some $n$, implying that our players will almost surely land in $O$.

$\endgroup$

You must log in to answer this question.

Start asking to get answers

Find the answer to your question by asking.

Ask question

Explore related questions

See similar questions with these tags.